Econ Exams 1-3

Pataasin ang iyong marka sa homework at exams ngayon gamit ang Quizwiz!

A 10 percent increase in the price of root beer causes a 5 percent increase in the quantity demanded of orange soda. This means that:​ a. ​root beer and orange soda are substitutes. b. ​the cross-price elasticity of demand is 1. c. ​the cross-price elasticity of demand is equal to 2. d. ​root beer and orange soda are complements. e. ​the cross-price elasticity of demand is equal to −2.

A

Assuming each firm produces the same product, rank the elasticity of demand for the FIRM from least to most elastic based on what type of market structure the firm competes in. a. monopoly, oligopoly, perfect competition b. perfect competition, oligopoly, monopoly c. oligopoly, monopoly, perfect competition d. perfect competition, oligopoly, monopoly e. they all have the same elasticity of demand since they sell the same product.

A

Compared to the 1970s, income in the United States today is a. less equally distributed. b. about the same in terms of equality. c. more equally distributed.

A

Economies of scale occur where _____ a. long-run average cost falls as one firm expands plant size. b. long-run average cost falls as new firms enter the industry. c. short-run average cost falls as one firm expands plant size. d. short-run average cost falls as new firms enter the industry. e. long-run average cost rises as one firm expands plant size.

A

Goods A and B are complements. An increase in the price of good A will cause __________. In this case the cross-price elasticity of demand between A and B is ____________ a. the demand curve for B to shift back; negative. b. the demand curve for B to shift out; negative. c. the demand curve for B to shift back; positive. d. a movement up the demand curve for B; negative. e. the demand curve for B to shift out; positive.

A

If a good is both rival and excludable, it is known as a(n)... a. private good. b. public good. c. club good (aka natural monopoly good). d. open access good. e. giffen good.

A

If the marginal social cost of producing a good exceeds the marginal private cost, then _____ a. the good produces a negative externality. b. the quantity of the good being produced is too low. c. the good produces a positive externality. d. social welfare is likely to increase by producing more of the good. e. the price charged for the good is too high.

A

In a perfectly competitive industry, what does an upward-sloping long run industry supply curve mean? a. That costs and prices rise as the industry grows. b. That the firm has diseconomies of scale. c. That long run average costs rise as one firm grows. d. That the firm has diminishing marginal product.

A

LeBron is a better point guard than Lonzo. Yet Lonzo will play point guard for the Lakers and LeBron will playsmall forward. What best describes this situation? ("point guard" and "small forward" are positions in basketball) a. LeBron has absolute advantage at point guard but Lonzo has comparative advantage at point guard. b. Lonzo has absolute and comparative advantage at point guard. c. LeBron has comparative advantage at point guard but Lonzo has absolute advantage at point guard. d. LeBron has comparative advantage at point guard but MORE comparative advantage at small forward.

A

Medicare is a(n) _____ a. social insurance program that provides health insurance to people aged 65 years and above, regardless of income. b. income assistance program that provides cash benefits to the elderly and the poor. c. social insurance program that provides health insurance only to people who suffer from work-related injuries or disabilities. d. social insurance program that provides health insurance to the poor depending on their income level e. income assistance program that supplements the wages of the working poor.

A

Other things constant, which of these is likely to increase the supply of vanilla ice cream? a. A decrease in the price of milk, an ingredient in ice cream b. An increase in the price of chocolate ice cream c. A decrease in the sales tax on ice cream d. An increase in the price of hot fudge e. An increase in the price of vanilla beans, an ingredient in ice cream

A

Refer to Exhibit 15.1 (Refer to Exam 3 Q24), which shows the cost and revenue curves for a natural monopolist—the operator of a subway system. To achieve the socially or "allocatively" efficient output rate, regulators could set the price at____ per trip. a. $0.50 b. $2.50 c. $1.50 d. $4.00 e. $1.25

A

Refer to Exhibit 8.4 (Refer to Exam 2 Q3), which shows the demand and the cost curves of a perfectly competitive firm. At a market price of P1, the profit-maximizing quantity for the firm is _____ a. d units of output. b. between d and e units of output. c. b units of output. d. a units of output. e. e units of output.

A

Refer to Exhibit 9.12 (Refer to Exam 2 Q13) which shows the cost and revenue curves faced by a profit-maximizing one-price monopolist. Deadweight loss caused by the monopoly is equal to a. dbc b. ap'd c. p'dbc d. apc e. apbd

A

Refer to Exhibit 9.12 (Refer to Exam 2 Q14), A perfectly competitive market would operate at point ____; while a monopolist would operate at point _____. a. c; d b. b; c c. d; c d. c; b e. c; a

A

Refer to Table 7.2 (Refer to Exam 2 Q5), which shows labor and the quantity of shoes produced by a firm. What is the marginal product of the third unit of labor? a. 25 pairs of shoes b. 50 pairs of shoes c. 75 pairs of shoes d. 45 pairs of shoes e. 15 pairs of shoes

A

Suppose Trostville (population 4) wants to put on a fireworks display. Levi would get $40 worth of benefit from it,Oz would get $30, Steve would get $20, and Erika would get $10. What is the socially efficient amount to spend on the display? a. $100, the sum of their benefits. b. $40, the maximum of their benefits. c. $25, the average of their benefits. d. $10, the minimum of their benefits. e. It's impossible to tell.

A

Suppose a restaurant has a monopoly in a certain small town. Its rent, which is one of the several fixed costs it incurs whether it sells food or not, has gone up. In the short run, the restaurant should _____ a. pay the higher rent and leave menu prices unchanged. b. pay the higher rent and lower menu prices. c. pay the higher rent and increase menu prices. d. shut down

A

When the consumption of a good generates positive externalities, _____ a. the private demand curve for the good lies below the marginal social benefit curve. b. the private demand curve for the good shifts leftward. c. the private demand curve for the good lies above the marginal social benefit curve. d. the equilibrium quantity of the good determined by the free private market is too high. e. the private demand curve for the good shifts rightward.

A

Which of the following represents a "command" or "centrally planned" aspect of the U.S. economy? a. The military. b. Amazon.com. c. Tipping at restaurants. d. Private universities.

A

A monopolist's demand curve is _____ a. the same as its average cost curve. b. identical to its market demand curve. c. its marginal revenue curve. d. the same as the demand curve faced by a firm in perfect competition. e. its marginal cost curve.

B

A wage offer that is set above the market wage and is intended to avoid the adverse selection problem is called a(n) _____ a. unionized wage. b. efficiency wage. c. minimum wage. d. indirect wage. e. equilibrium wage.

B

According to the Coase theorem, _____ a. it is necessary to assign property rights when bargaining costs are high. b. externality problems can be solved efficiently by the assignment of property rights if transaction costs are low. c. the assignment of property rights lead to equitable but inefficient solutions to externality problems. d. externality problems can be solved efficiently by the assignment of property rights if transaction costs are high. e. government regulation is necessary to eliminate negative externalities.

B

According to the graph in Figure 18-1 (Refer to Exam 3 Q12), the tax revenue collected will be which of the following? a. 14 million b. 1 million c. 1.4 million d. 12 million

B

At a given rate of output, marginal cost equals the slope of the _____ a. short-run average total cost curve. b. total cost curve. c. average variable cost curve. d. planning curve. e. long-run average cost curve.

B

Because of the ease of entry of new firms into monopolistically competitive markets in the long run, existing firms in these markets _____ a. can not earn economic profit in the short run. b. on average, earn no economic profit in the long run. c. charge a price equal to marginal cost. d. have no control over their price e. produce at the lowest average total cost.

B

Better production technology will... a. shift supply out, lower price, and cause demand to shift out. b. shift supply out, lower price, and increase quantity demanded. c. shift demand out, raise price, and increase quantity supplied. d. shift supply out, shift demand back and lower price. e. shift supply back, increase price, and decrease quantity demanded.

B

Compared to a firm in perfect competition, a monopolistically competitive firm tends to _____ a. produce the same quantity. b. produce less and charge a higher price. c. produce more and charge a higher price. d. produce more and charge a lower price. e. produce less and charge a lower price.

B

For a firm, "vertical integration" is synonymous with a. outsourcing. b. insourcing. c. collusion. d. economies of scope. e. horizontal mergers.

B

For a progressive tax system like ours, the marginal tax rate is _______ the average or "effective" tax rate. a. less than b. greater than c. equal to d. none of the above, the two rates cannot be compared.

B

For most products, the price elasticity of supply is likely to _____ as more time passes ("in the long run") a. get smaller b. get larger c. stay the same d. get more inelastic

B

If the current quantity in the market for hammers is inefficient, then it must be the case that a. producer surplus exceeds consumer surplus in the market for hammers. b. the sum of consumer surplus and producer surplus could be increased by moving to a different quantity. c. quantity supplied is equal to quantity demanded. d. consumer surplus exceeds producer surplus in the market for hammers. e. deadweight loss in the market must be zero

B

In game theory, what do you call a game in which both players have a dominant strategy but by both playing that strategy, players end up with a bad joint outcome? a. A tit-for-tat game. b. A prisoner's dilemma game. c. A Nash game d. A maximin game.

B

In order to increase society's well-being, measles vaccinations should be _____ a. subsidized to discourage production. b. subsidized to encourage production. c. taxed to encourage production. d. provided without government intervention. e. taxed to discourage production.

B

In the short run, increasing marginal costs are caused by a. increasing average costs. b. capacity constraints or fixed factors of production. c. limited variable inputs. d. diseconomies of scale. e. increasing marginal product.

B

Legislation that imposes costs on a few while bestowing benefits on many is known as a. special interest legislation. b. populist legislation. c. public goods legislation. d. competing interest legislation.

B

Mergers between firms in the same industry are called a. public mergers b. horizontal mergers c. Herfindahl mergers d. vertical mergers e. conglomerate mergers

B

My son knows that if he forgets to set his alarm in the morning, I will wake him up. As a result, he forgets to set his alarm more often. This is an example of what? a. the Tiebout hypothesis b. moral hazard c. rational ignorance d. the principal-agent problem e. adverse selection

B

Opportunity cost is defined as the:​ a. ​difference between the benefits from a choice and the costs of that choice. b. ​value of the best alternative not chosen. c. ​difference between the benefits from a choice and the benefits from the next best alternative. d. ​dollar cost of what is purchased. e. ​value of all alternatives not chosen.

B

Refer to Exhibit 7.4 (Refer to Exam 2 Q11), which shows short-run average cost curves. For output less than qa, when is average cost the lowest? a. at an output level less than q b. when the plant is small c. when the plant is medium-size d. when the plant is large e. at an output level more than qb

B

Refer to Exhibit 8.5 (Refer to Exam 2 Q30), which shows the demand and the cost curves of a perfectly competitive firm. At the profit-maximizing quantity, this firm will earn economic profit (or loss) equal to... a. zero b. -$12 c. $16 d. $12.60 e. -$16

B

Suppose Chipotle has a "half price" day (so prices falls 50%) and they sell 30% more burritos that day. How would you characterize the price elasticity of demand for burritos? Would total revenue be higher or lower that day? a. inelastic; higher b. inelastic; lower c. inelastic; can't tell d. elastic; higher e. elastic; lower

B

Suppose a price ceiling creates a shortage. Which of the following could possibly eliminate (or reduce) that shortage? a. an increase in consumer incomes (assume a normal good). b. a fall in input prices. c. setting the price ceiling to a lower price. d. a rise in input prices. e. an increase in the price of substitute goods.

B

Suppose the elasticity of demand for a good is 0.8 while the elasticity of supply is 1.2. What is true if a tax is placed on this good? a. all of the tax will be paid by the consumers of the good. b. most, but not all, of the tax will be paid by the consumers of the good. c. the producers and consumers of the good will each pay the same amount of the tax. d. most, but not all, of the tax will be paid by the producers of the good. e. all of the tax will be paid by the producers of the good.

B

Tariffs (taxes) on imported steel will help the US steel industry but the costs will be widespread as prices of things made with steel will rise. This is an example of: a. populist legislation. b. special interest legislation. c. public-interest legislation. d. the median-voter model. e. competing-interest legislation.

B

The "tradable permits" solution to pollution... a. imposes taxes on polluting firms. b. allows the market to determine which firms reduce pollution. c. is not as flexible as other approaches. d. requires well-defined property rights e. requires all firms to reduce pollution by the same amount.

B

The principal-agent problem is less likely to occur when getting a haircut than when getting a car repaired because ____ a. haircuts are less expensive than car repairs. b. there are no hidden actions when getting your hair cut. c. there is no principal-agent relationship in car repair. d. auto mechanics are less honest than hair stylists. e. there are no hidden actions when getting your car repaired.

B

What is an industry dominated by just a few firms? a. monopolistic competition b. an oligopoly c. a monopsony d. a monopoly e. perfectly competitive

B

When regulating a Natural Monopoly, why would the government choose to make the firm price at average cost rather than marginal cost? a. Because consumer surplus is highest under average cost pricing. b. Because average cost pricing does not require the government to subsidize the firm. c. Because it allows the firm to earn positive economic profit. d. Because average cost pricing achieves allocative efficiency.

B

Which of the following would not help identify market structure? a. the forms of competition among the firms in a market b. the price of a good sold in a market c. the ease of entry into a market d. the number of firms in a market e. the type of product produced in a market

B

"Economic Profit" will always be _______ "Accounting Profit". And the difference is ___________. a. less than; explicit costs b. greater than; implicit costs c. less than; implicit costs d. the same as; zero e. greater than; explicit costs

C

"Sunk costs" are... a. always to be taken into account when making a decision. b. costs that you can reduce (or "sink") if you choose wisely in the future. c. to be ignored when making a decision. d. the marginal cost of a choice or decision.

C

A stationary demand curve (one that is not shifting) can tell us... a. what would happen to quantity demanded if the price of a related good changes. b. what would happen to quantity demanded if consumers' incomes change. c. what would happen to quantity demanded if the price of the good changes. d. a, b, and c are all correct. e. nothing. It has to shift to give us any information.

C

An inferior good has ______. a. positive income elasticity of demand b. negative price elasticity of demand c. negative income elasticity of demand d. negative price elasticity of supply e. positive cross-price elasticity of demand

C

Barriers to entry _____ a. prevent the government from regulating a monopoly. b. prevent a natural monopoly from raising its price. c. may allow monopolies to earn profit in the long run. d. cause monopolies to experience diseconomies of scale in the long run. e. prevent monopolies from earning profit in the short run.

C

If a good has an income elasticity of demand equal to zero, it means that a. you will always buy zero regardless of your income b. the good is a luxury good. c. a change in your income won't affect how much you buy. d. you will always buy the same amount regardless of the price. e. if your income changes, your consumption of that product will drop to zero.

C

If marginal utility is negative, then total utility is... a. negative b. rising at a slower rate c. falling d. always positive

C

In a perfectly competitive market, what can one farmer do to change the market price? a. Sell more wheat. b. Sell less wheat. c. The firm cannot change the market price. d. Compete with the wheat market for sellers. e. Compete with neighboring farms for customers.

C

In game theory, a "tit-for-tat" strategy is a. when a player avoids the worst payoff on the board. b. when one player chooses the same strategy no matter what the other player does. c. when one player does whatever the other player did in the previous round of a repeated game. d. when a player chooses the same strategy as the other player in a one-shot game. e. when players always collude and choose the best outcome.

C

In the "Lemons Problem," because of ________, consumers are not willing to spend much for a car and as a result, _______ occurs and the good cars are taken off of the market. a. hidden characteristics; the winner's curse b. hidden actions; adverse selection c. hidden characteristics; adverse selection d. hidden characteristics; moral hazard e. hidden actions; moral hazard

C

Inferior goods are a. goods that you buy less of as the price falls. b. goods that are cheap. c. goods for which demand decreases as consumer income rises. d. goods that suck. e. goods that have a positive income elasticity of demand.

C

Recent tariffs have increased the price of steel in the U.S. At the same time, consumer incomes have been rising. Given this information (and realizing that cars are made from steel), what does the supply and demand model predict will happen to new car prices in the U.S.? a. Prices will fall. b. It is impossible to determine what will happen to prices without more info. c. Prices will rise. d. Prices will stay the same.

C

Refer to Exhibit 17.1 (Refer to Exam 3 Q2), which shows a market for electricity. to encourage the firm to produce the socially optimal amount, how much tax should be put on electricity? a. 10 cents per unit b. 15 cents per unit c. 5 cents per unit d. $0 - the firm will choose the socially optimal level without a tax. e. $25 per unit

C

Refer to Exhibit 8.9 (Refer to Exam 2 Q7), which shows a perfectly competitive firm's short-run output decisions.At price p2, the firm_____ a. has short-run losses less than the fixed cost. b. produces at a specific output to earn a normal profit. c. is indifferent between producing and shutting down. d. produces nothing. e. earns short-run economic profit by producing at a specific output.

C

Refer to Figure 7-21 (Refer to Exam 1 Q26). When the price is P1, area C represents a. profits. b. consumer surplus. c. producer surplus. d. total surplus.

C

Suppose households with income of $50,000 pay $5,000 in taxes, households with income of $100,000 pay$10,000 in taxes, and households with income of $300,000 pay $30,000 in taxes. Taxes in this example are a. progressive b. regressive c. proportional or "flat" d. none of the above.

C

Table 10.2 (Refer to Exam 3 Q3) depicts the payoff matrix facing Donny and KJ who are supposed to fight out back of the oldHenderson house after school. Each player has to choose whether to show up or not. What is the solution (Nash Equilibrium) to the game? (The first number is Donny's payoff, the second is KJ's) a. Donny doesn't show up but KJ does b. Donny shows up but KJ doesn't c. Both show up d. Both don't show up.

C

The following table (Refer to Exam 1 Q36) shows the marginal utility of pretzels for a consumer. The total utility derived from the consumption of three pretzels is _____ units. a. 10 b. 30 c. 60 d. 20 e. -10

C

To maximize utility, consumers should spend all income and consume in a manner that equates __________ for all good consumed. a. (total utility)/price b. marginal utility c. (marginal utility)/price d. money spent e. total utility

C

Transfer programs that do not tie benefits to contributions are called _____ a. compensation programs. b. health insurance programs. c. income assistance programs. d. social insurance programs. e. health aid programs.

C

A positive economic statement: a. is a statement of something good. b. is a statement of a true fact. c. is a statement of what ought to be, not what is. d. can be proven to be true or false.

D

All else equal, a good that is very important is likely to have ________ demand while a good with many substitutes is likely to have ____ demand. a. perfectly elastic; perfectly inelastic b. inelastic; inelastic. c. elastic; elastic d. inelastic; elastic e. elastic; inelastic

D

Chad wore pajama pants to the first two econ exams and aced them both. For the third exam, Chad decides that he doesn't have to study. All he has to do is wear his now "lucky" pajama pants. What mistake is Chad making? a. He is committing the fallacy of composition. b. He is not ignoring sunk costs. c. He is ignoring secondary effects. d. He is confusing association with causation.

D

Exhibit 10.5 (Refer to Exam 2 Q1) shows the demand, marginal revenue, and cost curves for a monopolistically competitive firm. At the profit-maximizing (or loss-minimizing) output and price, the firm would _____ a. have to expand to stay in business in the long run. b. be earning an economic profit. c. be better off shutting down, since total revenue does not cover fixed costs. d. be earning zero economic profit. e. be experiencing an economic loss.

D

For a firm, fixed costs... a. can always be avoided. b. exist in both the long and short run. c. rise as quantity rises. d. only exist in the short run. e. only exist in the long run.

D

If Helen can paint one room in the time it takes her to bake 40 cakes and Josh can paint one room in the time it takes him to bake 60 cakes, which of the following is definitely true? a. Helen has absolute advantage in painting. b. Josh has comparative advantage in painting. c. The opportunity cost of painting is lower for Josh. d. The opportunity cost of painting is lower for Helen. e. Helen's opportunity cost of painting one room is equal to 1/40 of a cake.

D

If a firm with a 10 percent market share merges with another firm with a 10 percent market share, and the other firms have market shares of 40 percent, 20 percent, 10 percent, and 10 percent, respectively, the Herfindahl-Hirschman Index (HHI) will _____ a. rise by 100. b. fall by 200. c. rise by 20. d. rise by 200. e. stay the same.

D

If the income Gini coefficient is falling, it means that a. income inequality is rising. b. the Lorenz curve is getting more bowed away from the line of equality. c. the Lorenz curve is shifting away from the line of equality. d. the Lorenz curve is getting less bowed away from the line of equality. e. income is falling for everyone.

D

If the price of a good increases and an individual's demand is elastic, identify the correct statement about total utility (TU), marginal utility (MU), consumer surplus (CS), and total expenditure (TE). (TE is the same as total revenue that we discussed in class). a. TU increases, MU increases, CS increases, and TE decreases. b. TU decreases, MU increases, CS decreases, and TE increases. c. TU decreases, MU decreases, CS decreases, and TE decreases. d. TU decreases, MU increases, CS decreases, and TE decreases. e. TU increases, MU decreases, CS decreases, and TE decreases.

D

In a means-tested program, a household qualifies for welfare benefits only if _____ a. the head of the household has a college degree. b. one of the members of the household is a war veteran. c. the family has dependent children. d. the household's income and assets are below a certain level. e. the head of the household is a single mother.

D

Refer to Exhibit 17.3 (Refer to Exam 3 Q35), which shows market equilibrium in the presence of an externality. While the market equilibrium is at point ___, the socially efficient price/quantity is at point __. a. b; d b. b; e c. b; c d. d; b e. d; e

D

Refer to Exhibit 9.2 (Refer to Exam 2 Q31). A non-discriminating, profit-maximizing monopolist will earn a profit of _____ per unit of output. a. $5 b. $0 c. $10 d. $4 e. $15

D

Refer to Exhibit 9.7 (Refer to Exam 2 Q8), which shows the cost and revenue curves faced by a profit-maximizing monopolist. If the monopolist engages in perfect price discrimination, its profit-maximizing output will be _____ a. 700 units. b. 976 units. c. 810 units. d. 884 units. e. 1,000 units.

D

Refer to Figure 21-3 (Refer to Exam 1 Q29). Which of the graphs in the figure reflects a decrease in the price of good Y only? a. graph a b. graph b c. graph c d. graph d

D

Refer to Table 9.3 (Refer to Exam 2 Q27). Marginal revenue from the fourth unit of output is _____ a. $3. b. $12. c. $4. d. $0. e. −$4.

D

Suppose a monopolistically competitive firm is producing at an output level where marginal revenue is less than marginal cost. This firm should _____ quantity and _____ price to increase profit or reduce losses. a. decrease; decrease b. increase, increase c. increase; decrease d. decrease; increase e. increase; not change

D

Suppose a perfectly competitive market were taken over and "monopolized" by one firm. Which of these would happen? a. Price would rise, quantity would fall, producer surplus would fall, and total surplus would fall. b. Price would fall, quantity would fall, consumer surplus would fall, and total surplus would fall. c. Price would rise, quantity would fall, consumer surplus would rise, and deadweight loss would fall. d. Price would rise, quantity would fall, consumer surplus would fall, and total surplus would fall. e. Price would fall, quantity would rise, consumer surplus would rise, and total surplus would fall.

D

To show that Chevy Camaros and Ford Mustangs are substitutes, one would calculate the ______ and get a______ number. a. price elasticity of demand for each car; positive b. income elasticity of demand for each car; positive c. cross-price elasticity; negative d. cross-price elasticity; positive e. price elasticity of demand for each car; negative

D

Which of the following is true of a perfectly competitive market? a. Firms face significant barriers to entry. b. Firms experience decreasing returns to scale. c. Each firm chooses the price at which it wants to sell its product. d. Each seller supplies only a small fraction of the total amount in a market. e. Firms experience constant returns to scale.

D

Which of the following statements about a demand curve is true? a. The demand curve for a good will not shift when the money income of consumers increases. b. If a supply curve shifts, thereby changing price, the demand curve will shift as well. c. If price increases, the demand curve will shift to the right. d. The demand curve for a good will not shift when price changes. e. If a demand curve shifts, the supply curve will shift as well irrespective of whether price changes.

D

Which of the graphs given below (Refer to Exam 1 Q25) best illustrates the impact on the PPF of a technological improvement that will make the use of the resources used to produce consumer goods more efficient but will not affect the production of capital goods? a. b b. a c. a and c d. c e. d

D

Which of these equalities is synonymous with "allocative efficiency"? a. P=AC b. MR=MC c. P=MR d. P=MC e. P=AVC

D

Who won the Nobel Prize in Economics in 2017 for his work in Behavioral Economics? a. Paul Krugman b. Richard Marx c. Jim Tiebout d. Richard Thaler e. Brad Daugherty

D

A "usury law" is a type of ______ imposed on ______________. a. price floor; wages for low-skill jobs b. price ceiling; used cars c. tax; poor people d. price floor; loanable funds e. price ceiling; interest rates

E

A increase in price from $6 to $10 leads to a fall in quantity demanded from 8 million to 4 million. Using the midpoint formula, what is the elasticity of demand? a. 3/2 b. 2/3 c. 1 d. 3/4 e. 4/3

E

Broccoli and cauliflower are substitutes. All else equal, an above average broccoli growing season will lead to... a. higher broccoli prices and higher cauliflower prices. b. lower broccoli prices and higher cauliflower prices. c. lower broccoli prices and higher quantities of both broccoli and cauliflower. d. lower broccoli prices and an increase in demand for cauliflower. e. lower broccoli prices and lower cauliflower prices.

E

Colluding oligopolistic firms, compared with competing oligopolistic firms, usually _____ a. welcome new firms. b. make it possible for consumers to pay lower prices. c. earn less economic profit. d. charge less. e. produce less.

E

If marginal cost exceeds average variable cost, then _____ a. marginal cost is greater than average total cost. b. average variable cost is decreasing. c. average variable cost is negative. d. average fixed cost is increasing. e. average variable cost is increasing.

E

If the unemployment rate rises, all else equal, what happens to the PPF? a. The PPF shifts inward. b. The PPF shifts outward. c. We move to a point closer to the PPF. d. We move to a point further outside the PPF. e. We move to a point further inside the PPF.

E

In a perfectly competitive, constant cost industry, starting in long run competitive equilibrium, a rise in demand will lead to _____ in the short run and _____ in the long run. a. higher prices and zero profits; higher prices and zero profits b. more firms; zero profit c. higher quantity; higher prices d. higher prices and positive profits; higher prices and zero profits e. higher prices and positive profits; more firms and zero profit

E

One example of "rent seeking" that we talked about is when a. a landlord bothers a tenant about the rent. b. consumers try to maximize utility. c. firms try to increase profits by becoming more efficient. d. a politician caters to the median voter. e. a group lobbies politicians to enact legislation that benefits that group.

E

Refer to Exhibit 17.1 (Refer to Exam 3 Q1), which shows a market for electricity. The total social gain from basing production on marginal social cost instead of marginal private cost is reflected by _____ (hint, it's the same as deadweight loss) a. 5 cents b. rectangle area abcd. c. 25 million kilowatt-hours. d. triangle cda e. triangle area abc.

E

Refer to Exhibit 7.3 (Refer to Exam 2 Q24), which shows the cost curves for a producer. A is the marginal cost curve (MC), B is the average variable cost curve (AVC), and C is the average total cost curve (ATC). When output is 10, then _____ a. fixed cost equals $7. b. marginal cost equals $10. c. fixed cost equals $10. d. variable cost equals $70. e. total cost equals $80.

E

Refer to Exhibit 9.14 (Refer to Exam 2 Q35), which shows the activity of a price-discriminating monopolist that divides its customers into two segments based on price elasticity of demand. How much will consumers with less price elasticity (more inelastic demand) pay? a. between 0 and $1.00 b. 0 c. $1.50 d. $1.00 e. $3.00

E

Suppose a soccer coach has been making $25,000 per year but gives up his coaching job in order to make soccer shoes. If his revenue from the sale of these shoes is $50,000 and his materials cost $20,000, then his economic profit is equal to _____ a. $80,000. b. $30,000. c. $50,000. d. $25,000. e. $5,000.

E

Suppose the price of a good falls. In order to move back into utility-maximizing "equilibrium", a consumer should a. consume more of that good to increase marginal utility b. buy more of other goods c. consume less of that good to decrease marginal utility d. consume less of that good to increase marginal utility. e. consume more of that good to decrease marginal utility

E

The "free rider" problem occurs when a good is a. not depletable. b. not available. c. not rival d. excludable e. not excludable.

E

The figure given below (Refer to Exam 1 Q3) depicts the milk market. The horizontal line, P, represents a price ceiling imposed by the government. Which of the following is true? a. The quantity demanded at the price ceiling will equal the quantity produced. b. There is a surplus when the price per gallon is $1. c. The equilibrium price would be $1 per unit without the price ceiling. d. In equilibrium, the quantity demanded is 800 gallons. e. The quantity supplied at the price ceiling is 500 gallons.

E


Kaugnay na mga set ng pag-aaral

MATH 4C Linear Algebra Chapter 2 TRUE OR FALSE

View Set

Project Management Ch3&4 Test Guide

View Set

Le grand Quiz de culture générale!

View Set

Give Me Liberty Chapter 24 Multiple Choice

View Set

تاريخ الفكر الاقتصادي

View Set

Economics Questions Chapter 34-36

View Set

RN Fundamentals Online Practice 2019 Test B

View Set